LSAT and Law School Admissions Forum

Get expert LSAT preparation and law school admissions advice from PowerScore Test Preparation.

 Administrator
PowerScore Staff
  • PowerScore Staff
  • Posts: 8917
  • Joined: Feb 02, 2011
|
#43114
Please post your questions below!
 yudeshen0624
  • Posts: 2
  • Joined: Jun 16, 2018
|
#46800
the right answer is C. But why?


It says “function PRIMARILY to”, but I don't think the final paragraph PRIMARILY discuss the implication of the author's research.
 Malila Robinson
PowerScore Staff
  • PowerScore Staff
  • Posts: 296
  • Joined: Feb 01, 2018
|
#46809
Hi yudeshen0624,
Answer C (discuss the implications of the author’s research) does explain the purpose of the last paragraph. The passage posits the hypotheses/views of "some cosmologists" against the hypothesis/view of the author & colleague. The final paragraph explains the hypothesized implications of the acceptance of the 'author & colleague's view' over the view of 'some cosmologists'.
It may help (our explanation & your understanding) to know why you don't think this was the primary function & what you thought the correct answer would be instead.
Hope this helps!
-Malila
 cathyli1996
  • Posts: 5
  • Joined: Jan 20, 2019
|
#62030
Dear Powerscore instructor,

For this question, I prephrase the answer to be: though the author's view is better than the cosmologists' view, the author's view does not invalidate the cosmologists' view, so basically the function of the last paragraph is to point out the two views are compatible!

With this prephrase in mind,I cannot relate it with "the implications of the author's research" in (C). Am I missing something?
 James Finch
PowerScore Staff
  • PowerScore Staff
  • Posts: 943
  • Joined: Sep 06, 2017
|
#62053
Hi Cathy,

It looks like the issue here is that your Prephrase is a little off. The way I read the passage was that the author was questioning the assumption that life requires the exact same "finely-tuned" physical laws that govern our universe, which are highly improbable, which leads to the conclusion that there must be an infinite number of other universes (multiverse) that enables our universe to exist by simple random chance. The research done by the author, mentioned in paragraph 4, challenges this assumption by showing that other sets of physical laws could also support the formation of matter and life.

This brings us to the paragraph in question: while the author clearly disagrees that with the assumption that our specific "finely tuned" physical laws are necessary for life, they also argue that this isn't evidence against the multiverse hypothesis, and strongly imply that the multiverse hypothesis is correct, for other reasons. This summary could be described, in very general terms, as "discussing the implications of the author's research;" effectively, the author wrote the last paragraph in order to clarify that their research didn't invalidate the multiverse hypothesis, despite invalidating what many may have seen as a necessary assumption for it.

Hope this clears things up!
 claudiagarin
  • Posts: 18
  • Joined: May 18, 2020
|
#76029
Hi, I really don't understand how the final paragraph is discussing the implications of the author's research? It looks like it is dismissing the suggestion that their findings go against the theory of a multiverse. could someone please explain?
 Adam Tyson
PowerScore Staff
  • PowerScore Staff
  • Posts: 5153
  • Joined: Apr 14, 2011
|
#76419
Thanks for asking, claudiagarin!. The bulk of the last paragraph is dedicated to answering the question posed by the author in the second sentence:
Do our findings therefore call the concept of the multiverse into question?
The answer, according to the author, is no, they do not, and she gives two reasons why not. That is discussion what the research does, or in this case does not, imply. It's an argument about what the research doesn't require, and "X does not imply Y" is still a discussion of the implications of X, even if the result is "no such implications."
User avatar
 bnlawyer98
  • Posts: 6
  • Joined: May 27, 2021
|
#88799
I am confused on how A isn't the correct answer. Isn't the author arguing that the view taken by some cosmologists that fine tuning is indirect evidence for the multiverse is an inadequate view and then the author goes on to explain why that view is inadequate.
User avatar
 Bob O'Halloran
PowerScore Staff
  • PowerScore Staff
  • Posts: 61
  • Joined: Jul 06, 2021
|
#88827
Hi Bnlawyer98
Thank you for your question.
The fine tuning argument is mentioned in the first sentence of the paragraph, but the focus of the paragraph is the second sentence-
"Do our findings therefore call the concept of the multiverse into question?"
Then the rest of the paragraph goes on to answer that question, no longer referring to fine tuning.
Hope this helps and let us know if you have any other questions
Bob
 dimi.wassef@yahoo.com
  • Posts: 34
  • Joined: Aug 26, 2021
|
#92927
What is wrong with
B : indicate the kinds of questions to which the author’s research can be extended ?

Get the most out of your LSAT Prep Plus subscription.

Analyze and track your performance with our Testing and Analytics Package.